Anda di halaman 1dari 20

PURPLE COMET!

MATH MEET April 2013 HIGH SCHOOL - SOLUTIONS Copyright c Titu Andreescu and Jonathan Kane

Problem 1
Two years ago Tom was 25% shorter than Mary. Since then Tom has grown 20% taller, and Mary has grown 4 inches taller. Now Mary is 20% taller than Tom. How many inches tall is Tom now? Answer: 45 Note that 25% less means or
6 5 1 4

less or

3 4

as much. Also 20% more means

1 5

more

as much. Let Toms and Marys current heights in inches be t and m,


t
6 5

respectively. Then, two years ago their heights would have been

and m 4,

6 respectively. The relationship between their heights today gives 5 t = m, and

the relationship between their heights two years ago gives


5450 45 t,

t
6 5

=3 4 (m 4).
3 4

5 Substituting m in the rst equation for m in the second gives 6 t=

6 5t

4 .

Solving yields

4 3

5 6t

6 5t

4 and 4 =

6 5

10 9

t=

so t =

445 4

= 45.

Problem 2
The following diagram shows an eight-sided polygon ABCDEF GH with side lengths 8, 15, 8, 8, 8, 6, 8, and 29 as shown. All of its angles are right angles. Turn this eight-sided polygon into a six-sided polygon by connecting B to D with an edge and E to G with an edge to form polygon ABDEGH . Find the perimeter of ABDEGH .

Answer: 80 By the Pythagorean Theorem BD = 152 + 82 = 289 = 17 and EG = 82 + 62 = 100 = 10. Thus, the requested perimeter is 8 + 17 + 8 + 10 + 8 + 29 = 80.

Problem 3
In how many rearrangements of the numbers 1, 2, 3, 4, 5, 6, 7, 8, 9 do the numbers form a hill, that is, the numbers form an increasing sequence at the beginning up to a peak, and then form a decreasing sequence to the end such as in 129876543 or 258976431? Answer: 254 Since in any such arrangement the number 9 must be the number at the peak, the numbers in the arrangement must increase to the position of the 9, and then decrease after that position. The sequence is then completely determined by the numbers that come before the 9. There are 28 = 256 subsets of {1, 2, 3, 4, 5, 6, 7, 8} that could make up the numbers that come before the 9, except that neither the empty set nor the entire set result in an arrangement that both increases up to 9 and decreases after the 9. Therefore, there are 256 2 = 254 possible sequences that form a hill. 2

Problem 4
The greatest common divisor of n and 180 is 12. The least common multiple of n and 180 is 720. Find n. Answer: 48 For any positive integers a and b, a b = gcd(a, b) lcm(a, b). Thus, n=
12720 180

= 12 4 = 48.

Problem 5
How many four-digit positive integers have exactly one digit equal to 1 and exactly one digit equal to 3? Answer: 720 An acceptable four-digit number could begin with a 1 or a 3. Then there are two ways to choose whether the rst digit is a 1 or 3, three ways to choose the position of the other digit (1 or 3), and then 8 8 = 64 ways to select the other two digits. This means that there are 2 3 64 = 384 ways to choose such a number which begins with 1 or 3. An acceptable four-digit number could begin with a digit other than 1 or 3. Then there are 3 ways to choose where the digit 1 can be, and 2 ways to choose where the digit 3 can be. After that there are 7 ways to select a digit to appear in the rst position (zero is not acceptable), and 8 ways to select a digit to go into the remaining unselected position. This means that there are 3 2 7 8 = 336 ways to choose such a number which does not begin with 1 or 3. Thus, there are 384 + 336 = 720 ways to choose an acceptable number.

Problem 6
In four years Kay will be twice as old as Gordon. Four years after that Shaun will be twice as old as Kay. Four years after that Shaun will be three times as old as Gordon. How many years old is Shaun now?

Answer: 48 Let the current age of Gordon, Kay, and Shaun be x, y , and z , respectively. Then the conditions in the problem imply that 2(x + 4) = y + 4, 2(y + 8) = z + 8, and 3(x + 12) = z + 12. These equations simplify to 2x + 4 = y , 2y + 16 = z + 8 = 2(2x + 4) + 16 = 4x + 24, and 3x + 24 = z so 4x + 24 8 = 3x + 24 and x = 8. It follows that y = 2 8 + 4 = 20 and z = 4 8 + 16 = 48. The requested age of Shaun is z = 48.

Problem 7
Find the least six-digit palindrome that is a multiple of 45. Note that a palindrome is a number that reads the same forward and backwards such as 1441 or 35253. Answer: 504405 A number is a multiple of 45 exactly when it is a multiple both 5 and 9. The decimal representation of a number divisible by 5 must end in the digit 0 or 5. A six-digit palindrome cannot end in 0. A number is a multiple of 9 if its digits add to a multiple of 9. A six-digit palindrome whose digits add to a multiple of 9 must have its digits add to a multiple of 18 since each of its digits appears twice. The least six-digit palindrome that satises these requirements must, therefore, begin with 5 which is followed by two digits adding to 4. The least such palindrome is 504405.

Problem 8
Naomi has three colors of paint which she uses to paint the pattern below. She paints each region a solid color, and each of the three colors is used at least once. If Naomi is willing to paint two adjacent regions with the same color, how many color patterns could Naomi paint?

Answer: 540 Because the pattern has 6 regions, and Naomi could select any of the three colors for each region, there are 36 = 729 ways to color the pattern using either 1, 2, or 3 colors. There are 3 ways to color the pattern using just 1 color. There are 3(26 2) = 186 ways to color the pattern using exactly 2 colors. Thus, there are 729 186 3 = 540 patterns that use all three colors.

Problem 9
|5x2 2 5 | |x 8| if and only if x is in the interval [a, b]. There are relatively prime positive integers m and n so that b a = Answer: 18 Consider two cases: x 8 0 and x 8 < 0. If x 8 0, then |x 8| = x 8 and (x 8) 5x2 5x x +
2 38 5 2 5 m n.

Find m + n.

x 8. The second inequality simplies to

0 which has no solutions. If x 8 < 0, then |x 8| = 8 x and


2 5

(8 x) 5x2

8 x. The rst inequality simplies to 0 5x2 x +

38 5

which is valid for all real numbers x. The second inequality simplies to 5 x2 + x
42 5

0 or 25x2 + 5x 42 0. Factoring gives (5x + 7)(5x 6) 0

7 6 which shows that the inequality is satised for all x in [ 5 , 5 ]. The required

dierence is

6 5

7 5 =

13 5 ,

and the requested sum is 13 + 5 = 18.

Problem 10
The number N is the product of two primes. The sum of the positive divisors of N that are less than N is 2014. Find N . Answer: 4022 Let p and q be prime numbers so that N = pq . The positive integer divisors of N are 1, p, and q , so 2014 = 1 + p + q and p + q = 2013. Clearly, one of the primes must be even and, thus, must be 2. The two primes are 2 and 2011, so N = 2 2011 = 4022.

Problem 11
After Jennifer walked r percent of the way from her home to the store, she turned around and walked home, got on her bicycle, and bicycled to the store and back home. Jennifer bicycles two and a half times faster than she walks. Find the largest value of r so that returning home for her bicycle was not slower than her walking all the way to and from the store without her bicycle. Answer: 60 Let the distance from Jennifers home to the store be s, and her walking speed be w. Then the time it would have taken Jennifer to walk to the store and home again is
2s w.

The time it took Jennifer to walk r percent of the way to


2sr 100w

the store, return home, and then bicycle to and from the store is

2s 2.5w .

Set these two travel times equal to each other and solve for r to get
2s w

2sr 100w

2s 2.5w

or 1 =

r 100

1 2.5

and r = 60.

Problem 12
How many four-digit positive integers have no adjacent equal even digits? For example, count numbers such as 1164 and 2035 but not 6447 or 5866. Answer: 7801 There are 9999 999 = 9000 four-digit positive integers. Let A be the set of four-digit integers where the rst two digits are the same even digit, let B be the set of four-digit integers where the second and third digits are the same

even digit, and let C be the set of four-digit integers where the last two digits are the same even digit. The requested answer is 9000 |A B C |. The Inclusion-Exclusion Principle can be used to nd |A B C | = (|A| + |B | + |C |) (|A B | + |A C | + |B C |) + |A B C | = (4 100 + 5 90 + 5 90) (4 10 + 4 5 + 5 9) + 4 = 1300 105 + 4 = 1199. The requested count is 9000 1199 = 7801.

Problem 13
There are relatively prime positive integers m and n so that

m = log4 32log9 27 . n
Find m + n. Answer: 19

log4 32log9 27 = log9 27 log4 32 =


The requested sum is 15 + 4 = 19.

log3 27 log3 9

log2 32 log2 4

3 2

5 2

15 . 4

Problem 14
How many ordered triples (a, b, c) of positive integers satisfy a b c and a b c = 1000? Answer: 19 If positive integers a, b, and c satisfy a b c = 1000, then a = 2m1 5n1 , b = 2m2 5n2 , and c = 2m3 5n3 where the mi and ni are non-negative integers satisfying m1 + m2 + m3 = n1 + n2 + n3 = 3. The number of solutions to m1 + m2 + m3 = 3 in non-negative integers is the same as the number of ways of distributing 3 indistinguishable items to three containers. The well-known stars and bars counting technique (3 stars with 3 1 = 2 bars) shows that the number of such solutions is
3+31 31 5 2

= 10. Thus, the number of solutions

for the mi and ni is 10 10 = 100, although only some of these solutions satisfy a b c. There is exactly one of these 100 solutions where a = b = c and that is a = b = c = 10. There are nine of the 100 solutions where exactly two 7

of the a, b, and c are equal. They are a = b = 1 and c = 1000, a = b = 2 and c = 250, a = b = 5 and c = 40, and their permutations. The other 100 1 9 = 90 solutions consist of three distinct numbers. Now considering only solutions satisfying a b c, there is one with all three numbers equal,
9 3

= 3 with exactly two of the numbers equal, and

90 6

= 15 with three distinct

numbers. This accounts for 1 + 3 + 15 = 19 solutions. Alternatively, since there are only 19 triples (a, b, c), it is not hard to list all of them. First note that a cannot exceed 3 1000 = 10. Then for each possible divisor, a = 1, 2, 4, 5, 8, 10, one can easily identify the divisors b of satisfying a b value of c =
1000 ab . 1000 a . 1000 a

Each such (a, b) combination results in a unique

Problem 15
For what value of x does the following determinant have the value 2013? 5+x 1 2 Answer: 497 The determinant expands as (5 + x)(1 + x)2 + 8 3 2 + (2 + x)1 1 2(1 + x)(2 + x) 1 3(5 + x) 2 1 8 = (2x2 + 12x + 10 + 48 + 2 + x) (2x2 + 6x + 4 + 15 + 3x + 16) = 25 + 4x. This equals 2013 when x = 497. 8 1+x 1 2+x 3 2

Problem 16
The gure below shows a 90 90 square with each side divided into three equal segments. Some of the endpoints of these segments are connected by straight lines. Find the area of the shaded region.

Answer: 3540

Place the square on coordinate plane so that the vertices of the square are (0, 0), (90, 0), (90, 90), and (0, 90). Label the four intersection points inside the square A, B , C , and D as shown. Note that the shaded region is the region of the square not contained in any of the twelve numbered triangles. Each of the numbered triangles has an edge along an edge of the square and a vertex at one of the intersection points A, B , C , or D. Thus, once the coordinates of the four intersection points are known, it will be easy to calculate the areas of the twelve triangles and, consequently, the area of the shaded region. Point A is at the intersection of the line y = 1 2 x + 60 and the line y = 2x + 30. Solving these two equations simultaneously gives A = (20, 70). Point B is at
2 the intersection of the line y = 3 x + 30 and the line y = 3 2 x. Solving these two

equations simultaneously gives B = (36, 54). By symmetry C = (54, 36) and D = (70, 20). For j = 1, 2, 3, . . . , 12, let Aj be the area of the triangle labeled j . Then 30 20 = 300, 2 30 20 A2 = = 300, 2 30 36 = 540. A3 = 2 A1 = By symmetry A4 = A3 , A5 = A2 , and A6 = A1 . Also by symmetry, for j = 1, 2, 3, 4, 5, 6, Aj = A13j . It follows that the total area of the numbered triangles is 4(300 + 300 + 540) = 4560. Since the area of the entire square is 90 90 = 8100, the area of the shaded region is 8100 4560 = 3540.

Problem 17
For positive integers m and n, the decimal representation for the fraction begins 0.711 followed by other digits. Find the least possible value for n. Answer: 45 Note some obvious candidates: 0.711000... = and 0.711111... =
7 10 711 1000 , m n

0.711711711... =

711 999

79 111 ,

1 90

64 90

32 45 .

To show that 45 is the smallest possible

denominator, consider the continued fraction representation for

0.7115 =

1 1+
1 2+
1 2+ 1 6+ 1+ 1 1

.
8+ 1 1 1+ 3

The rst partial convergent of this continue fraction that is within 0.005 of 0.7115 is 0.7111 . . . = 1 1+
1 2+
1 2+ 1 6

32 . 45

This shows that 45 is the least positive integer denominator for a fraction with values between 0.7110 and 0.712. 10

Problem 18
Two concentric circles have radii 1 and 4. Six congruent circles form a ring where each of the six circles is tangent to the two circles adjacent to it as shown. The three lightly shaded circles are internally tangent to the circle with radius 4 while the three darkly shaded circles are externally tangent to the circle with radius 1. The radius of the six congruent circles can be written
k+ m , n

where k , m, and n are integers with k and n relatively prime. Find

k + m + n.

Answer: 126 Let B be the center of the concentric circles. Let A and C be the centers of two adjacent darkly and lightly shaded circles, respectively. Let D be the point where BC intersects the circle with radius 4 as shown. Let r be the radius of the six congruent circles.

11

Note that since the centers of the six congruent circles are equally spaced, ABC is 60 . By the Law of Cosines, AB 2 + BC 2 2(AB )(BC ) cos 60 = AC 2 or (1 + r)2 + (4 r)2 2(1 + r)(4 r) 1 = (2r)2 . 2

This simplies to r2 + 9r 13 = 0. The equation has positive solution 9+ 92 4(13) r= = 9+2 133 . The requested sum is 9 + 133 + 2 = 126. 2

Problem 19
There is a pile of eggs. Joan counted the eggs, but her count was o by 1 in the 1s place. Tom counted the eggs, but his count was o by 1 in the 10s place. Raoul counted the eggs, but his count was o by 1 in the 100s place. Sasha, Jose, Peter, and Morris all counted the eggs and got the correct count. When these seven people added their counts together, the sum was 3162. How many eggs were in the pile? Answer: 439 If all seven people had found the correct count, the sum of the seven counts would have been a multiple of 7. But 3162 5 (mod 7). Note that 1 1 (mod 7), 10 3 (mod 7), and 100 2 (mod 7). Because of the errors introduced by the counts of Joan, Tom, and Raoul, the sum is 3162 5 1 3 2 (mod 7). Of the eight possible assignments of plus or minus in this sum, only 1 3 + 2 = 2 5 (mod 7) gives a result that is 5 (mod 7). Thus, it must be that Joans count was 1 too low, Toms count was 10 too low, and Raouls count was 100 too high. The corrected total is 3162 + 1 + 10 100 = 3073 = 7 439. The number of eggs in the pile is 439.

Problem 20
1 Let z be a complex number satisfying (z + z )(z + 1 z

+ 1) = 1. Evaluate

(3z 100 +

2 z 100

+ 1)(z 100 +

2 z 100

+ 3).

12

Answer: 36
1 Multiply (z + z )(z + 1 z

+ 1) = 1 by z 2 to get (z 2 + 1)(z 2 + z + 1) = z 2 which


z 5 1 z 1

reduces to z 4 + z 3 + z 2 + z + 1 = 0 or
20

= 0. Thus, z satises the given

relation if z is a fth root of unity not equal to 1. This implies that z 100 = z 5 = 1, and (3z 100 +
2 z 100

+ 1)(z 100 +

2 z 100

+ 3) =

(3 + 2 + 1)(1 + 2 + 3) = 36.

Problem 21
Evaluate (2 sec2 1 )(2 sec2 2 )(2 sec2 3 ) (2 sec2 89 ). Answer: 0 Since sec 45 =
1 cos 45

=
2

1
1 2

2, the product includes the factor

2 sec2 45 = 2

2 = 0, and the product is 0.

Problem 22
Find the least three-digit number that is equal to the sum of its digits plus twice the product of its digits. Answer: 397 Suppose the three-digit number abc is equal to the sum of its digits plus twice the product of its digits. Then a + b + c + 2a b c = 100a + 10b + c so 2a b c = 99a + 9b. Note that since a + b + c + 2a b c 100, none of the digits can be zero. It follows that a must divide 9b, and b must divide 99a. From 9b = a(2b c 99) it follows that 2b c > 99, so bc must be at least 50. Since c cannot exceed 9, b must be at least 6. If b = 6, then c must be 9 in order for bc to be at least 50. Then solving 2a b c = 99a + 9b for a yields a = 6. So one candidate is abc = 669. If b = 7, then since b must divide 99a, a must be 7 and abc would exceed 669 and does not need to be considered further. If b = 8, then since b must divide 99a, a must be 8, and again, this does not need to be considered further. If b = 9, then since a must divide 9b, a must be either 1, 3, or 9 and c =
11a+9 2a .

Again, a = 9 is not of interest. If

13

a = 1, c =

11+9 2

= 10 which is not possible. If a = 3, c =

113+9 23

= 7 resulting

in abc = 397. This is the least three-digit number that is equal to the sum of its digits plus twice the product of its digits.

Problem 23
The diagram below shows the regular hexagon BCEGHJ surrounded by the rectangle ADF I . Let be the measure of the acute angle between the side EG of the hexagon and the diagonal of the rectangle AF . There are relatively prime positive integers m and n so that sin2 =
m n.

Find m + n.

Answer: 55

Let the intersection of AF and EG be K , let L be the point on CD so that LK is parallel to DF , and assume without loss of generality that ABJ is a 30 60 90 triangle, BC = CE = EG = GH = HJ = JB = 2, and DE = EF = IJ = JA = 3. AB = CD = F G = HI = 1. Because By the Pythagorean Theorem AF = AD2 + DF 2 = 42 + (2 3)2 = 28 = 2 7. Thus, cos AKL = cos AF D =
3 7

and sin AKL =

2 . 7

Note that EKL = 30 . The required


1 2

value is sin EKA = sin(EKL + LKA) = sin(EKL) cos(LKA) + cos(EKL) sin(LKA) =


33 2 7

3 7

3 2

2 7

27 28 .

The requested sum is 27 + 28 = 55. 14

Problem 24
Find the remainder when 333333 is divided by 33. Answer: 27 By Fermats Little Theorem, 333333 [333 (mod 11)][333
(mod 10)]

33

5 (mod 11). This means that when 333333 is divided by 11, the remainder is 5, so when 333333 is divided by 33 = 3 11, the remainder must be 5, 5 + 11 = 16, or 5 + 22 = 27. Since 333333 is clearly a multiple of 3, its remainder when divided by 33 must also be a multiple of 3. Thus, the correct remainder is 27.

Problem 25
In how many ways can you write 12 as an ordered sum of integers where the smallest of those integers is equal to 2? For example, 2 + 10, 10 + 2, and 3 + 2 + 2 + 5 are three such ways. Answer: 70 For k = 2, 3, 4, 5, 6, calculate the number of ways a sequence of k integers can have a sum of 12 where the smallest of the integers is 2. Suppose that k integers, all at least 2, add to 12. Subtracting 2 from each of the integers results in k non-negative integers which add to 12 2k where at least one of the integers is 0. The number of ways to add k non-negative integers to equal 12 2k is
(122k)+(k1) k1 11k k 1

. Alternatively, the number of ways for k


112k k 1

positive integers to add to 12 2k is

. Thus, the number of ways k


11k k1

integers can be added to 12 where the smallest integer is 2 is This gives k=2: k=3: k=4: k=5: k=6:
9 1 8 2 7 3 6 4 5 5

112k k 1

7 1 5 2 3 3 1 4 1 5

= 9 7 = 2, = 28 10 = 18, = 35 1 = 34, = 15 0 = 15, = 1 0 = 1.

The required number of sequences is 2 + 18 + 34 + 15 + 1 = 70.

15

Problem 26
The diagram below shows the rst three gures of a sequence of gures. The rst gure shows an equilateral triangle ABC with side length 1. The leading edge of the triangle going in a clockwise direction around A is labeled AB and is darkened in on the gure. The second gure shows the same equilateral triangle with a square with side length 1 attached to the leading clockwise edge of the triangle. The third gure shows the same triangle and square with a regular pentagon with side length 1 attached to the leading clockwise edge of the square. The fourth gure in the sequence will be formed by attaching a regular hexagon with side length 1 to the leading clockwise edge of the pentagon. The hexagon will overlap the triangle. Continue this sequence through the eighth gure. After attaching the last regular gure (a regular decagon), its leading clockwise edge will form an angle of less than 180 with the side AC of the equilateral triangle. The degree measure of that angle can be written in the form Find m + n.
m n

where m and n are relatively prime positive integers.

Answer: 1088 The degree measure of the internal angle of a regular k sided gure is Thus, the angle in question is 180 60 + 90 + 108 + 120 +
900 7 1 3 180(k2) . k

2 4

3 5

4 6

+ +

8 10

4 + 135 + 140 + 144 = 925 7 . Subtracting this from 1081 7 .

3 360 = 1080 yields 154 3 7 =

The requested sum is 1081 + 7 = 1088.

16

Problem 27
Suppose a, b, and c are real numbers that satisfy a + b + c = 5 and
1 a

1 b

1 c

1 =5 . Find the greatest possible value of a3 + b3 + c3 .

Answer: 125 Note that if a, b, and c satisfy the given equations, then (a 5)(b 5)(c 5) = abc 5(bc + ac + ab) + 25(a + b + c) 125 =
1 + abc[1 5( a 1 b 1 +1 c )] + 25 5 125 = abc[1 5 5 ] = 0. Thus, at least one of a,

b, or c must be 5, and since a + b + c = 5, the other two numbers must add to zero. Thus, there is a number x so that a3 + b3 + c3 = 53 + x3 + (x)3 = 125, and the greatest possible value for a3 + b3 + c3 is 125.

Problem 28
Let A, B, C, D, E, F, G, H be the eight vertices of a 30 30 30 cube as shown. The two gures ACF H and BDEG are congruent regular tetrahedra. Find the volume of the intersection of these two tetrahedra.

Answer: 4500 The volume of the cube is 303 = 27,000. The tetrahedron AEF H has height AE = 30 and base
3030 2

EF H which is a right isosceles triangle with area


30450 3

= 450. Thus, tetrahedron AEF H has volume

= 4500. Since the

tetrahedron ACF H can be obtained by starting with the cube and removing four tetrahedra congruent to AEF H , the volume of ACF H is 27,000 4 4500 = 9000. This is also the volume of the congruent tetrahedron BDEG. Let I , J , and K be the midpoints of the line segments AF , AH , and F H , respectively. Then the region inside tetrahedron AEF H and outside

17

tetrahedron BDEG is made up of the three congruent tetrahedra EAIJ , EF IK , and EHJK . The rst of these tetrahedra, for example, is EAIJ with base volume AEI with area
15225 3 3030 4

= 225, and height

30 2

= 15. Thus, EAIJ has

= 1125. It follows that the volume of the region inside AEF H

and outside BDEG is 3 1125 = 3375, and the volume inside the cube and outside both ACF H and CDEG is 4 3375 = 13,500. Then the volume of the union of ACF H and CDEG is 27,000 13,500 = 13,500. But this can also be written |ACF H | + |CDEG| |ACF H CDEG| so 13,500 = 9000 + 9000 |ACF H CDEG|, and the desired volume of the intersection is |ACF H CDEG| = 9000 + 9000 13,500 = 4500. Alternatively, the desired intersection can be seen to be the convex region bounded by the eight planes of the eight sides of the two tetrahedra. Thus, the desired intersection is a regular octahedron. Each of the eight planes can be seen to be a distance
15 3

from the center of the cube. It follows that the

octahedron itself is a union of eight right tetrahedra with four sides equal to 15 and height 8
15 6
3

15 3

showing that the volume of the desired intersection is

= 4500.

Problem 29
You can tile a 2 5 grid of squares using any combination of three types of tiles: single unit squares, two side by side unit squares, and three unit squares in the shape of an L. The diagram below shows the grid, the available tile shapes, and one way to tile the grid. In how many ways can the grid be tiled?

18

Answer: 798 Let Ak be the number of ways to tile a 2 k grid of squares. Let Bk be the number of ways to tile a 2 k grid with the rst square in one row missing. Note that A0 = 1 (because the only way to tile a 2 0 grid is by using zero tiles), A1 = 2, B0 = 0, and B1 = 1. On a 2 k grid there are nine ways to ll the leftmost grid column with tiles. The possibilities are shown in the diagram below.

After lling in the rst column with one of these tiling patterns, the remaining squares of the 2 k grid must be tiled. It follows that Ak+1 = 2Ak + 3Ak1 + 4Bk . Similarly, on a 2 k grid with one square missing in the leftmost column, there are three ways to tile the lone square in the rst column by using either a single square tile, a double square tile, or an L shaped tile. It follows that Bk+1 = Ak + Bk + Ak1 . From these recursive equations, it is not hard to build a table of values for Ak and Bk . k 0 1 2 3 4 5 Ak 1 2 11 44 189 798 Bk 0 1 4 17 72 305

The requested number of tilings is 798.

19

Problem 30
Suppose x, y , and z are integers that satisfy the system of equations x2 y + y 2 z + z 2 x = 2186 xy 2 + yz 2 + zx2 = 2188. Evaluate x2 + y 2 + z 2 .

Answer: 245 Suppose x, y , and z are integers that satisfy the system of equations. Then (x y )(y z )(z x) = (xyz + xy 2 + yz 2 + zx2 ) (xyz + x2 y + y 2 z + z 2 x) = 2188 2186 = 2. Because x y , y z , and z x are all integers, these three dierences must all be either 1 or 2. The only way this can happen is for x, y , and z to be three consecutive integers. Without loss of generality, assume x = y 1 and z = y + 1. Then 2188 = (y 1)y 2 + y (y + 1)2 + (y + 1)(y 1)2 = (y 3 y 2 ) + (y 3 + 2y 2 + y ) + (y 3 y y 2 + 1) = 3y 3 + 1. So y 3 =
21881 3

= 729

and y = 9. Hence x = 8, z = 10, and the requested sum is 82 + 92 + 102 = 245.

20

Anda mungkin juga menyukai